Bytelearn - cat image with glassesAI tutor

Welcome to Bytelearn!

Let’s check out your problem:

Let 
g be a continuous function on the closed interval 
[-1,4], where 
g(-1)=-4 and 
g(4)=1.
Which of the following is guaranteed by the Intermediate Value Theorem?
Choose 1 answer:
(A) 
g(c)=3 for at least one 
c between -4 and 1
(B) 
g(c)=3 for at least one 
c between -1 and 4
(C) 
g(c)=-3 for at least one 
c between -1 and 4
(D) 
g(c)=-3 for at least one 
c between -4 and 1

Let g g be a continuous function on the closed interval [1,4] [-1,4] , where g(1)=4 g(-1)=-4 and g(4)=1 g(4)=1 .\newlineWhich of the following is guaranteed by the Intermediate Value Theorem?\newlineChoose 11 answer:\newline(A) g(c)=3 g(c)=3 for at least one c c between 4-4 and 11\newline(B) g(c)=3 g(c)=3 for at least one c c between 1-1 and 44\newline(C) g(c)=3 g(c)=-3 for at least one c c between 1-1 and 44\newline(D) g(c)=3 g(c)=-3 for at least one c c between 4-4 and 11

Full solution

Q. Let g g be a continuous function on the closed interval [1,4] [-1,4] , where g(1)=4 g(-1)=-4 and g(4)=1 g(4)=1 .\newlineWhich of the following is guaranteed by the Intermediate Value Theorem?\newlineChoose 11 answer:\newline(A) g(c)=3 g(c)=3 for at least one c c between 4-4 and 11\newline(B) g(c)=3 g(c)=3 for at least one c c between 1-1 and 44\newline(C) g(c)=3 g(c)=-3 for at least one c c between 1-1 and 44\newline(D) g(c)=3 g(c)=-3 for at least one c c between 4-4 and 11
  1. Introduction: The Intermediate Value Theorem states that if a function is continuous on a closed interval [a,b][a, b] and NN is any number between f(a)f(a) and f(b)f(b), then there exists at least one cc in the interval [a,b][a, b] such that f(c)=Nf(c) = N. We are given that gg is continuous on the interval [1,4][-1, 4], g(1)=4g(-1) = -4, and NN00.
  2. IVT Explanation: We need to determine which of the given statements is guaranteed by the Intermediate Value Theorem. Let's analyze each option:\newline(A) g(c)=3g(c) = 3 for at least one cc between 4-4 and 11. This option is not relevant because the interval for cc should be between the xx-values 1-1 and 44, not the yy-values 4-4 and 11.
  3. Option Analysis: (B) g(c)=3g(c) = 3 for at least one cc between 1-1 and 44. Since g(1)=4g(-1) = -4 and g(4)=1g(4) = 1, and 33 is between 4-4 and 11, the Intermediate Value Theorem guarantees that there is at least one cc in the interval cc00 such that g(c)=3g(c) = 3.
  4. Option A: (C)g(c)=3(C) g(c) = -3 for at least one cc between 1-1 and 44. Since g(1)=4g(-1) = -4 and g(4)=1g(4) = 1, and 3-3 is between 4-4 and 11, the Intermediate Value Theorem guarantees that there is at least one cc in the interval cc00 such that cc11.
  5. Option B: (D)g(c)=3(D) g(c) = -3 for at least one cc between 4-4 and 11. This option, like option (A)(A), is not relevant because the interval for cc should be between the xx-values 1-1 and 44, not the yy-values 4-4 and 11.
  6. Option C: Comparing options (B) and (C), we see that both are correct according to the Intermediate Value Theorem. However, the question asks us to choose one answer. Since the options in the problem statement might be exclusive, there might be an error in the problem statement or the options provided. In a typical scenario, only one of these options would be correct based on the values given for g(1)g(-1) and g(4)g(4). If we assume that the problem statement is correct and only one answer is to be chosen, we would need additional information to determine which of the two is the correct answer.

More problems from Intermediate Value Theorem